Download as pdf or txt
Download as pdf or txt
You are on page 1of 61

Calculus 1

Chapter 5. Integrals
5.3. The Definite Integral—Examples and Proofs

October 28, 2020

() Calculus 1 October 28, 2020 1 / 38


Table of contents
1 Exercise 5.3.6
2 Example 5.3.1. A Non-Integrable Function
3 Theorem 5.2. Rules Satisfied by Definite Integrals
4 Exercise 5.3.10
5 Exercise 5.3.63
6 Example 5.3.A
7 Exercise 5.3.65
8 Exercise 5.3.36
9 Exercise 5.3.18
10 Exercise 5.3.62
11 Exercise 5.3.76
12 Exercise 5.3.88
() Calculus 1 October 28, 2020 2 / 38
Exercise 5.3.6

Exercise 5.3.6

n q
X
Exercise 5.3.6. Express the limit lim 4 − ck2 ∆xk , where P is a
kPk→0
k=1
partition of [0, 1], as a definite integral.

Solution. With P = {x0 , x1 , . . . , xn } a partition√of [a, b] = [0, 1],


ck ∈ [xk−1 , xk ], ∆xk = xk − xk−1 , and f (x) = 4 − x 2 we have that
n q
X Z b Z 1p
lim 2
4 − ck ∆xk = f (x) dx = 4 − x 2 dx . 
kPk→0 a 0
k=1

() Calculus 1 October 28, 2020 3 / 38


Exercise 5.3.6

Exercise 5.3.6

n q
X
Exercise 5.3.6. Express the limit lim 4 − ck2 ∆xk , where P is a
kPk→0
k=1
partition of [0, 1], as a definite integral.

Solution. With P = {x0 , x1 , . . . , xn } a partition√of [a, b] = [0, 1],


ck ∈ [xk−1 , xk ], ∆xk = xk − xk−1 , and f (x) = 4 − x 2 we have that
n q
X Z b Z 1p
lim 2
4 − ck ∆xk = f (x) dx = 4 − x 2 dx . 
kPk→0 a 0
k=1

() Calculus 1 October 28, 2020 3 / 38


Example 5.3.1. A Non-Integrable Function

Example 5.3.1

1, if x is rational
Example 5.3.1. Show that the function f (x) =
0, if x is irrational
is not Riemann integrable over the interval [0, 1].
Solution. If f is Riemann integrable on [0, 1] then by the definition of
Z 1 n
X
definite integral, f (x) dx = lim f (ck ) ∆xk for any choice of
0 kPk→0
k=1
ck ∈ [xk−1 , xk ]. Now in any interval [xk−1 , xk ] there are both rational and
irrational numbers.

() Calculus 1 October 28, 2020 4 / 38


Example 5.3.1. A Non-Integrable Function

Example 5.3.1

1, if x is rational
Example 5.3.1. Show that the function f (x) =
0, if x is irrational
is not Riemann integrable over the interval [0, 1].
Solution. If f is Riemann integrable on [0, 1] then by the definition of
Z 1 n
X
definite integral, f (x) dx = lim f (ck ) ∆xk for any choice of
0 kPk→0
k=1
ck ∈ [xk−1 , xk ]. Now in any interval [xk−1 , xk ] there are both rational and
irrational numbers. So we can choose each ck to be rational in which case
Z 1 X n
each f (ck ) = 1 and f (x) dx = lim f (ck ) ∆xk =
0 kPk→0
k=1
n
X n
X
lim (1) ∆xk = lim ∆xk = lim 1 = 1 since the sum of the
kPk→0 kPk→0 kPk→0
k=1 k=1
length of the subintervals is the length of [0, 1] (namely 1).

() Calculus 1 October 28, 2020 4 / 38


Example 5.3.1. A Non-Integrable Function

Example 5.3.1

1, if x is rational
Example 5.3.1. Show that the function f (x) =
0, if x is irrational
is not Riemann integrable over the interval [0, 1].
Solution. If f is Riemann integrable on [0, 1] then by the definition of
Z 1 n
X
definite integral, f (x) dx = lim f (ck ) ∆xk for any choice of
0 kPk→0
k=1
ck ∈ [xk−1 , xk ]. Now in any interval [xk−1 , xk ] there are both rational and
irrational numbers. So we can choose each ck to be rational in which case
Z 1 X n
each f (ck ) = 1 and f (x) dx = lim f (ck ) ∆xk =
0 kPk→0
k=1
n
X n
X
lim (1) ∆xk = lim ∆xk = lim 1 = 1 since the sum of the
kPk→0 kPk→0 kPk→0
k=1 k=1
length of the subintervals is the length of [0, 1] (namely 1).

() Calculus 1 October 28, 2020 4 / 38


Example 5.3.1. A Non-Integrable Function

Example 5.3.1 (continued)


1, if x is rational
Example 5.3.1. Show that the function f (x) =
0, if x is irrational
is not Riemann integrable over the interval [0, 1].

Solution (continued). We can also choose each ck to be irrational in


Z 1 n
X
which case each f (ck ) = 0 and f (x) dx = lim f (ck ) ∆xk =
0 kPk→0
k=1
n
X n
X
lim (0) ∆xk = lim 0 = lim 0 = 0. But we cannot have both
kPk→0 kPk→0 kPk→0
k=1 k=1
Z 1 Z 1
f (x) dx = 1 and f (x) dx = 0, so f is not Riemann integrable over
0 0
[0, 1]. 

() Calculus 1 October 28, 2020 5 / 38


Example 5.3.1. A Non-Integrable Function

Example 5.3.1 (continued)


1, if x is rational
Example 5.3.1. Show that the function f (x) =
0, if x is irrational
is not Riemann integrable over the interval [0, 1].

Solution (continued). We can also choose each ck to be irrational in


Z 1 n
X
which case each f (ck ) = 0 and f (x) dx = lim f (ck ) ∆xk =
0 kPk→0
k=1
n
X n
X
lim (0) ∆xk = lim 0 = lim 0 = 0. But we cannot have both
kPk→0 kPk→0 kPk→0
k=1 k=1
Z 1 Z 1
f (x) dx = 1 and f (x) dx = 0, so f is not Riemann integrable over
0 0
[0, 1]. 

() Calculus 1 October 28, 2020 5 / 38


Theorem 5.2. Rules Satisfied by Definite Integrals

Theorem 5.2
Theorem 5.2. Rules Satisfied by Definite Integrals. Suppose f and g
are integrable over the interval [a, b]. Then:
Z b Z b
3. Constant Multiple: kf (x) dx = k f (x) dx
a a
4. Sum
Z b and Difference: Z b Z b
(f (x) ± g (x)) dx = f (x) dx ± g (x) dx
a a a
6. Max-Min Inequality: If max f and min f are the maximum and
minimum values of f on [a, b], then
Z b
min f · (b − a) ≤ f (x) dx ≤ max f · (b − a).
a

Z b Z b
7. Domination: f (x) ≥ g (x) on [a, b] ⇒ f (x) dx ≥ g (x) dx.
a a

() Calculus 1 October 28, 2020 6 / 38


Theorem 5.2. Rules Satisfied by Definite Integrals

Theorem 5.2 (continued 1)


Theorem 5.2. Rules Satisfied by Definite Integrals. Suppose f and g
are integrable over the interval [a, b]. Then:
Z b Z b
3. Constant Multiple: cf (x) dx = c f (x) dx.
a a
n
X
Proof. Let P be a partition of [a, b] and let f (ck ) ∆xk be an
k=1
Z b n
X
associated Riemann sum. Then f (x) dx = lim f (ck ) ∆xk and
a kPk→0
k=1
Z b n
X
cf (x) dx = lim cf (ck ) ∆xk
a kPk→0
k=1
Xn
= lim c f (ck ) ∆xk since multiplication
kPk→0
k=1
distributes over addition
() Calculus 1 October 28, 2020 7 / 38
Theorem 5.2. Rules Satisfied by Definite Integrals

Theorem 5.2 (continued 2)


Theorem 5.2. Rules Satisfied by Definite Integrals. Suppose f and g
are integrable over the interval [a, b]. Then:
Z b Z b
3. Constant Multiple: cf (x) dx = c f (x) dx.
a a

Proof (continued). . . .
Z b n
X
cf (x) dx = lim c f (ck ) ∆xk
a kPk→0
k=1
n
X
= c lim f (ck ) ∆xk by the Constant Multiple Rule,
kPk→0
k=1
Theorem 2.1(3)
Z b
= c f (x) dx,
a

as claimed.
() Calculus 1 October 28, 2020 8 / 38
Theorem 5.2. Rules Satisfied by Definite Integrals

Theorem 5.2 (continued 3)


Theorem 5.2. Rules Satisfied by Definite Integrals. Suppose f and g
are integrable over the interval [a, b]. Then:
4. Sum
Z b and Difference: Z b Z b
(f (x) ± g (x)) dx = f (x) dx ± g (x) dx.
a a a
n
X
Proof (continued). Let P be a partition of [a, b] and let f (ck ) ∆xk
k=1
n
X
and g (ck ) ∆xk be associated Riemann sums. Then by definition
k=1
Rb Pn
f (x) dx = limkPk→0 f (ck ) ∆xk and
Rab Pk=1
n
a g (x) dx = limkPk→0 k=1 g (ck ) ∆xk , so
Z b Z b n
X n
X
f (x) dx ± g (x) dx = lim f (ck ) ∆xk ± lim g (ck ) ∆xk
a a kPk→0 kPk→0
k=1 k=1

() Calculus 1 October 28, 2020 9 / 38


Theorem 5.2. Rules Satisfied by Definite Integrals

Theorem 5.2 (continued 4)


Proof (continued). . . .
Z b Z b n
X n
X
f (x) dx ± g (x) dx = lim f (ck ) ∆xk ± lim g (ck ) ∆xk
a a kPk→0 kPk→0
k=1 k=1

n n
!
X X
= lim f (ck ) ∆xk ± g (ck ) ∆xk
kPk→0
k=1 k=1
by the Sum and Difference Rules, Theorem 2.1(1 and 2)
n
!
X
= lim (f (ck ) ∆xk ± g (ck ) ∆xk )
kPk→0
k=1
by commutivity and addition and subtraction
n
!
X
= lim (f (ck ) ± g (ck )) ∆xk since multiplication
kPk→0
k=1
distributes over addition
() Calculus 1 October 28, 2020 10 / 38
Theorem 5.2. Rules Satisfied by Definite Integrals

Theorem 5.2 (continued 5)


Theorem 5.2. Rules Satisfied by Definite Integrals. Suppose f and g
are integrable over the interval [a, b]. Then:
4. Sum
Z b and Difference: Z b Z b
(f (x) ± g (x)) dx = f (x) dx ± g (x) dx.
a a a

Proof (continued). . . .
n
!
Z b Z b X
f (x) dx ± g (x) dx = lim (f (ck ) ± g (ck )) ∆xk
a a kPk→0
k=1
Z b
= (f (x) ± g (x)) dx,
a

n
!
Z b X
since (f (x) ± g (x)) dx = lim (f (ck ) ± g (ck )) ∆xk , by
a kPk→0
k=1
definition.
() Calculus 1 October 28, 2020 11 / 38
Theorem 5.2. Rules Satisfied by Definite Integrals

Theorem 5.2 (continued 6)

Theorem 5.2. Rules Satisfied by Definite Integrals. Suppose f and g


are integrable over the interval [a, b]. Then:
6. Max-Min Inequality: If max f and min f are the maximum and
minimum values of f on [a, b], then
Z b
min f · (b − a) ≤ f (x) dx ≤ max f · (b − a).
a

n
X
Proof (continued). Let P be a partition of [a, b] and let f (ck ) ∆xk
k=1
be
R b an associated Riemann
Pn sum. Then by definition
a f (x) dx = limkPk→0 k=1 f (ck ) ∆xk . Notice that
Xn
min f ≤ f (ck ) ≤ max f for all ck ∈ [a, b] and ∆xk = (b − a).
k=1

() Calculus 1 October 28, 2020 12 / 38


Theorem 5.2. Rules Satisfied by Definite Integrals

Theorem 5.2 (continued 7)


Proof (continued). So we have
Z b Xn n
X
f (x) dx = lim f (ck ) ∆xk ≥ lim min f ∆xk
a kPk→0 kPk→0
k=1 k=1
n
X
= lim min f ∆xk since multiplication
kPk→0
k=1
distributes over addition
Xn
= min f lim ∆xk by the Constant Multiple Rule,
kPk→0
k=1
Theorem 2.1(3)
n
X
= min f lim (b − a) since ∆xk = b − a
kPk→0
k=1
= min f · (b − a),
as claimed.
() Calculus 1 October 28, 2020 13 / 38
Theorem 5.2. Rules Satisfied by Definite Integrals

Theorem 5.2 (continued 8)


Proof (continued). Similarly,
Z b X n n
X
f (x) dx = lim f (ck ) ∆xk ≤ lim max f ∆xk
a kPk→0 kPk→0
k=1 k=1
n
X
= lim max f ∆xk since multiplication
kPk→0
k=1
distributes over addition
Xn
= max f lim ∆xk by the Constant Multiple Rule,
kPk→0
k=1
Theorem 2.1(3)
n
X
= max f lim (b − a) since ∆xk = b − a
kPk→0
k=1
= max f · (b − a),
as claimed.
() Calculus 1 October 28, 2020 14 / 38
Theorem 5.2. Rules Satisfied by Definite Integrals

Theorem 5.2 (continued 9)


Theorem 5.2. Rules Satisfied by Definite Integrals. Suppose f and g
are integrable over the interval [a, b]. Then:
Z b Z b
7. Domination: f (x) ≥ g (x) on [a, b] ⇒ f (x) dx ≥ g (x) dx.
a a

n
X
Proof (continued). Let P be a partition of [a, b] and let f (ck ) ∆xk
k=1
n
X
and g (ck ) ∆xk be associated Riemann sums. Then by definition
k=1
Rb Pn
f (x) dx = limkPk→0 f (ck ) ∆xk and
Rab Pk=1
n
a g (x) dx = limkPk→0 k=1 g (ck ) ∆xk . Since f (x) ≥ g (x) on [a, b] then
f (ck ) ≥ g (ck ) for all ck ∈ [a, b], and so
Z b X n n
X Z b
f (x) dx = lim f (ck ) ∆xk ≥ lim g (ck ) ∆xk = g (x) dx. 
a kPk→0 kPk→0 a
k=1 k=1
() Calculus 1 October 28, 2020 15 / 38
Exercise 5.3.10

Exercise 5.3.10
Exercise
Z 9 5.3.10. Suppose
Z 9 that f is h are
Z integrable and that
9
f (x) dx = −1, f (x) dx = 5, and h(x) dx = 4. Use the rules in
1 7 Z 9 7 Z
9
Theorem 5.2 to find (a) −2f (x) dx, (b) (f (x) − h(x)) dx,
Z 9 1 Z 1 7 Z
7
(c) (2f (x) − 3h(x)) dx, (d) f (x) dx, (e) f (x) dx, and
Z 77 9 1

(f) (h(x) − f (x)) dx.


9

Solution. (a) We have


Z 9 Z 9
−2f (x) dx = −2 f (x) dx by Constant Multiple Rule, Thm 5.2(3)
1 1
Z 9
= −2(−1) = 2 since f (x) dx = −1. 
1

() Calculus 1 October 28, 2020 16 / 38


Exercise 5.3.10

Exercise 5.3.10
Exercise
Z 9 5.3.10. Suppose
Z 9 that f is h are
Z integrable and that
9
f (x) dx = −1, f (x) dx = 5, and h(x) dx = 4. Use the rules in
1 7 Z 9 7 Z
9
Theorem 5.2 to find (a) −2f (x) dx, (b) (f (x) − h(x)) dx,
Z 9 1 Z 1 7 Z
7
(c) (2f (x) − 3h(x)) dx, (d) f (x) dx, (e) f (x) dx, and
Z 77 9 1

(f) (h(x) − f (x)) dx.


9

Solution. (a) We have


Z 9 Z 9
−2f (x) dx = −2 f (x) dx by Constant Multiple Rule, Thm 5.2(3)
1 1
Z 9
= −2(−1) = 2 since f (x) dx = −1. 
1

() Calculus 1 October 28, 2020 16 / 38


Exercise 5.3.10

Exercise 5.3.10 (continued 1)


Exercise
Z 9 5.3.10. Suppose
Z 9 that f is h are
Z integrable and that
9
f (x) dx = −1, f (x) dx = 5, and h(x) dx = 4. Use the rules in
1 7 Z 9 7

Theorem 5.2 to find (b) (f (x) − h(x)) dx.


7

Solution (continued). (b) We have


Z 9 Z 9 Z 9
(f (x) − h(x)) dx = f (x) dx − h(x) dx
7 7 7
by the Difference Rule, Theorem 5.2(4)
Z 9
= (5) − (4) = 1 since f (x) dx = 5
7
Z 9
and h(x) dx = 4. 
7

() Calculus 1 October 28, 2020 17 / 38


Exercise 5.3.10

Exercise 5.3.10 (continued 2)


Exercise
Z 9 5.3.10. Suppose
Z 9 that f is h are
Z integrable and that
9
f (x) dx = −1, f (x) dx = 5, and h(x) dx = 4. Use the rules in
1 7 Z 9 7

Theorem 5.2 to find (c) (2f (x) − 3h(x)) dx.


7
Z 9
Solution (continued). (c) We have (2f (x) − 3h(x)) dx =
7
Z 9 Z 9
= 2f (x) dx + −3h(x) dx by the Sum Rule, Theorem 5.2(4)
7 7
Z 9 Z 9
= 2 f (x) dx − 3
h(x) dx by Constant Mult. Theorem 5.2(3)
7 7
Z 9 Z 9
= 2(5) − 3(4) = −2 since f (x) dx = 5 and h(x) dx = 4. 
7 7

() Calculus 1 October 28, 2020 18 / 38


Exercise 5.3.10

Exercise 5.3.10 (continued 3)

Exercise
Z 9 5.3.10. Suppose
Z 9 that f is h are
Z integrable and that
9
f (x) dx = −1, f (x) dx = 5, and h(x) dx = 4. Use the rules in
1 7 Z 1 7

Theorem 5.2 to find (d) f (x) dx.


9

Solution (continued). (d) We have


Z 1 Z 9
f (x) dx = − f (x) dx by the Order of Integration, Theorem 5.2(1)
9 1
Z 9
= −(−1) = 1 since f (x) dx = −1. 
1

() Calculus 1 October 28, 2020 19 / 38


Exercise 5.3.10

Exercise 5.3.10 (continued 4)

Exercise
Z 9 5.3.10. Suppose
Z 9 that f is h are
Z integrable and that
9
f (x) dx = −1, f (x) dx = 5, and h(x) dx = 4. Use the rules in
1 7 Z 7 7

Theorem 5.2 to find (e) f (x) dx.


1

Solution (continued). (e) By Additivity (Theorem 5.2(5)) we have


Z 7 Z 9 Z 9
f (x) dx + f (x) dx = f (x) dx, then
Z1 7 Z7 9 Z1 9
f (x) dx = f (x) dx − f (x) dx. So
Z1 7 1 7 Z 9
f (x) dx = (−1) − (5) = −6 , since f (x) dx = −1 and
Z1 9 1

f (x) dx = 5. 
7

() Calculus 1 October 28, 2020 20 / 38


Exercise 5.3.10

Exercise 5.3.10 (continued 5)


Exercise
Z 9 5.3.10. Suppose
Z 9 that f is h are
Z integrable and that
9
f (x) dx = −1, f (x) dx = 5, and h(x) dx = 4. Use the rules in
1 7 Z 7 7

Theorem 5.2 to find (f) (h(x) − f (x)) dx.


9
Z 7
Solution (continued). (f) We have (h(x) − f (x)) dx =
9
Z 7 Z 7
= h(x) dx − f (x) dx by the Difference Rule, Theorem 5.2(4)
9 9
Z 9 Z 9
= − f (x) dx by Order of Integration, Theorem 5.2(1)
h(x) dx +
7 7
Z 9 Z 9
= −(4) + (5) = 1 since f (x) dx = 5 and h(x) dx = 4. 
7 7

() Calculus 1 October 28, 2020 21 / 38


Exercise 5.3.63

Exercise 5.3.63
Z b
Exercise 5.3.63. Let c be a constant. Prove that c dx = c(b − a).
a

Proof. Let f (x) = c. Then f is continuous on [a, b] so, by “Integrability of


Continuous Functions” (Theorem 5.1), f is integrable on [a, b]. Therefore,
we can consider any sequence of partitions which have a norm approaching
0. So we consider an equal width partition P = {x0 , x1 , . . . , xn } for which
∆xk = ∆x = (b − a)/n, xk = a + k(b − a)/n, and ck ∈ [xk−1 , xk ] (see
Note 5.3.A).

() Calculus 1 October 28, 2020 22 / 38


Exercise 5.3.63

Exercise 5.3.63
Z b
Exercise 5.3.63. Let c be a constant. Prove that c dx = c(b − a).
a

Proof. Let f (x) = c. Then f is continuous on [a, b] so, by “Integrability of


Continuous Functions” (Theorem 5.1), f is integrable on [a, b]. Therefore,
we can consider any sequence of partitions which have a norm approaching
0. So we consider an equal width partition P = {x0 , x1 , . . . , xn } for which
∆xk = ∆x = (b − a)/n, xk = a + k(b − a)/n, and ck ∈ [xk−1 , xk ] (see
Note 5.3.A). Now kPk = ∆x = (b − a)/n, so when n → ∞ we have
kPk → 0. So the value of the Riemann integral is given by
Z b n   n  
X b−a X b−a
c dx = lim f (ck ) = lim c
a n→∞ n n→∞ n
k=1 k=1
 
b−a
= lim nc by Theorem 5.2.A(4)
n→∞ n
= lim c(b − a) = c(b − a).
n→∞
() Calculus 1 October 28, 2020 22 / 38
Exercise 5.3.63

Exercise 5.3.63
Z b
Exercise 5.3.63. Let c be a constant. Prove that c dx = c(b − a).
a

Proof. Let f (x) = c. Then f is continuous on [a, b] so, by “Integrability of


Continuous Functions” (Theorem 5.1), f is integrable on [a, b]. Therefore,
we can consider any sequence of partitions which have a norm approaching
0. So we consider an equal width partition P = {x0 , x1 , . . . , xn } for which
∆xk = ∆x = (b − a)/n, xk = a + k(b − a)/n, and ck ∈ [xk−1 , xk ] (see
Note 5.3.A). Now kPk = ∆x = (b − a)/n, so when n → ∞ we have
kPk → 0. So the value of the Riemann integral is given by
Z b n   n  
X b−a X b−a
c dx = lim f (ck ) = lim c
a n→∞ n n→∞ n
k=1 k=1
 
b−a
= lim nc by Theorem 5.2.A(4)
n→∞ n
= lim c(b − a) = c(b − a).
n→∞
() Calculus 1 October 28, 2020 22 / 38
Example 5.3.A

Example 5.3.A
Example 5.3.A. Use a regular partition of [a, b] with ck = xk to prove
Z b
b 2 a2
that for a < b: x dx = − .
a 2 2
Proof. Let f (x) = x. Then f is continuous on [a, b] so, by “Integrability
of Continuous Functions” (Theorem 5.1), f is integrable on [a, b].
Therefore, we can consider any sequence of partitions which have a norm
approaching 0. So we consider an equal width partition
P = {x0 , x1 , . . . , xn } for which ∆xk = ∆x = (b − a)/n,
xk = a + k(b − a)/n, and ck ∈ [xk−1 , xk ] satisfies ck = xk = a + k(b − a)/n
(see Note 5.3.A).

() Calculus 1 October 28, 2020 23 / 38


Example 5.3.A

Example 5.3.A
Example 5.3.A. Use a regular partition of [a, b] with ck = xk to prove
Z b
b 2 a2
that for a < b: x dx = − .
a 2 2
Proof. Let f (x) = x. Then f is continuous on [a, b] so, by “Integrability
of Continuous Functions” (Theorem 5.1), f is integrable on [a, b].
Therefore, we can consider any sequence of partitions which have a norm
approaching 0. So we consider an equal width partition
P = {x0 , x1 , . . . , xn } for which ∆xk = ∆x = (b − a)/n,
xk = a + k(b − a)/n, and ck ∈ [xk−1 , xk ] satisfies ck = xk = a + k(b − a)/n
(see Note 5.3.A). Now kPk = ∆x = (b − a)/n, so when n → ∞ we have
kPk → 0. So the value of the Riemann integral is given by
b n   n  
b−a b−a
Z X X
x dx = lim f (ck ) = lim ck
a n→∞ n n→∞ n
k=1 k=1

() Calculus 1 October 28, 2020 23 / 38


Example 5.3.A

Example 5.3.A
Example 5.3.A. Use a regular partition of [a, b] with ck = xk to prove
Z b
b 2 a2
that for a < b: x dx = − .
a 2 2
Proof. Let f (x) = x. Then f is continuous on [a, b] so, by “Integrability
of Continuous Functions” (Theorem 5.1), f is integrable on [a, b].
Therefore, we can consider any sequence of partitions which have a norm
approaching 0. So we consider an equal width partition
P = {x0 , x1 , . . . , xn } for which ∆xk = ∆x = (b − a)/n,
xk = a + k(b − a)/n, and ck ∈ [xk−1 , xk ] satisfies ck = xk = a + k(b − a)/n
(see Note 5.3.A). Now kPk = ∆x = (b − a)/n, so when n → ∞ we have
kPk → 0. So the value of the Riemann integral is given by
b n   n  
b−a b−a
Z X X
x dx = lim f (ck ) = lim ck
a n→∞ n n→∞ n
k=1 k=1

() Calculus 1 October 28, 2020 23 / 38


Example 5.3.A

Example 5.3.A (continued 1)


Proof (continued).
b n   n   
b−a b−a b−a
Z X X
x dx = lim ck = lim a+k
a n→∞ n n→∞ n n
k=1 k=1
n n
  X !
b−a b−aX
= lim a+ k
n→∞ n n
k=1 k=1
   
b−a b − a n(n + 1)
= lim (na) +
n→∞ n n 2
n
X n(n + 1)
since k=
2
k=1
!
b − a 2 n(n + 1)
  
= lim (b − a)a +
n→∞ n 2

() Calculus 1 October 28, 2020 24 / 38


Example 5.3.A

Example 5.3.A (continued 2)


Proof (continued). 2  !
Z b 
b−a n(n + 1)
x dx = lim (b − a)a +
a n→∞ n 2
n(n + 1)
= (b − a)a + (b − a)2 lim
n→∞ 2n2 
n + n 1/n2
2

2
= (b − a)a + (b − a) lim
n→∞ 2n2 1/n2
1 + 1/n
= (b − a)a + (b − a)2 lim
n→∞ 2
1 + limn→∞ 1/n
= (b − a)a + (b − a)2
2
1 + (0) b 2 − 2ab + a2
= (b − a)a + (b − a)2 = ab − a2 +
2 2
b 2 a 2
= − . 
2 2
() Calculus 1 October 28, 2020 25 / 38
Exercise 5.3.65

Exercise 5.3.65
Exercise 5.3.65. Use a regular partition of [a, b] with ck = xk to prove
Z b
b 3 a3
that for a < b: x 2 dx = − .
a 3 3
Proof. Let f (x) = x 2 . Then f is continuous on [a, b] so, by “Integrability
of Continuous Functions” (Theorem 5.1), f is integrable on [a, b].
Therefore, we can consider any sequence of partitions which have a norm
approaching 0. So we consider an equal width partition
P = {x0 , x1 , . . . , xn } for which ∆xk = ∆x = (b − a)/n,
xk = a + k(b − a)/n, and ck ∈ [xk−1 , xk ] satisfies ck = xk = a + k(b − a)/n
(see Note 5.3.A).

() Calculus 1 October 28, 2020 26 / 38


Exercise 5.3.65

Exercise 5.3.65
Exercise 5.3.65. Use a regular partition of [a, b] with ck = xk to prove
Z b
b 3 a3
that for a < b: x 2 dx = − .
a 3 3
Proof. Let f (x) = x 2 . Then f is continuous on [a, b] so, by “Integrability
of Continuous Functions” (Theorem 5.1), f is integrable on [a, b].
Therefore, we can consider any sequence of partitions which have a norm
approaching 0. So we consider an equal width partition
P = {x0 , x1 , . . . , xn } for which ∆xk = ∆x = (b − a)/n,
xk = a + k(b − a)/n, and ck ∈ [xk−1 , xk ] satisfies ck = xk = a + k(b − a)/n
(see Note 5.3.A). Now kPk = ∆x = (b − a)/n, so when n → ∞ we have
kPk → 0. So the value of the Riemann integral is given by
b n   n  
b−a b−a
Z X X
2
x dx = lim f (ck ) = lim ck2
a n→∞ n n→∞ n
k=1 k=1

() Calculus 1 October 28, 2020 26 / 38


Exercise 5.3.65

Exercise 5.3.65
Exercise 5.3.65. Use a regular partition of [a, b] with ck = xk to prove
Z b
b 3 a3
that for a < b: x 2 dx = − .
a 3 3
Proof. Let f (x) = x 2 . Then f is continuous on [a, b] so, by “Integrability
of Continuous Functions” (Theorem 5.1), f is integrable on [a, b].
Therefore, we can consider any sequence of partitions which have a norm
approaching 0. So we consider an equal width partition
P = {x0 , x1 , . . . , xn } for which ∆xk = ∆x = (b − a)/n,
xk = a + k(b − a)/n, and ck ∈ [xk−1 , xk ] satisfies ck = xk = a + k(b − a)/n
(see Note 5.3.A). Now kPk = ∆x = (b − a)/n, so when n → ∞ we have
kPk → 0. So the value of the Riemann integral is given by
b n   n  
b−a b−a
Z X X
2
x dx = lim f (ck ) = lim ck2
a n→∞ n n→∞ n
k=1 k=1

() Calculus 1 October 28, 2020 26 / 38


Exercise 5.3.65

Exercise 5.3.65 (continued 1)


Proof (continued).
n n 
b−a 2 b−a
Z b     
X b−a X
x 2 dx = lim ck2 = lim a+k
a n→∞ n n→∞ n n
k=1 k=1
  n  2 !
b−a X 2 b−a 2 b−a
= lim a + 2ak +k
n→∞ n n n
k=1
n  n !
b−a 2X 2
  
b−a 2 b−aX
= lim (na ) + 2a k+ k
n→∞ n n n
k=1 k=1
   
b−a b − a n(n + 1)
= lim (na2 ) + 2a
n→∞ n n 2
 2  !
b−a n(n + 1)(2n + 1)
+
n 6
since k=1 k = n(n+1) and nk=1 k 2 = n(n+1)(2n+1)
Pn P
2 6

() Calculus 1 October 28, 2020 27 / 38


Exercise 5.3.65

Exercise 5.3.65 (continued 2)


Proof (continued).
Z b    
2 b−a 2 b − a n(n + 1)
x dx = lim (na ) + 2a
a n→∞ n n 2
 2  !
b−a n(n + 1)(2n + 1)
+
n 6
  
2 b − a n(n + 1)
= lim (b − a) a + 2a 2
n→∞ n 2
2
 
(b − a) n(n + 1)(2n + 1)
+
n3 6
 2
(n + n)/n2
 
2
= lim (b − a) a + 2a(b − a)
n→∞ 2
3 2 3
 
(2n + 3n + n)/n
+(b − a)2
6

() Calculus 1 October 28, 2020 28 / 38


Exercise 5.3.65

Exercise 5.3.65 (continued 3)


Proof (continued).
Z b  2
(n + n)/n2
 
x 2 dx = lim (b − a) a2 + 2a(b − a)
a n→∞ 2
3 2 3
 
(2n + 3n + n)/n
+(b − a)2
6
  
2 1 + 1/n
= lim (b − a) a + 2a(b − a)
n→∞ 2
2
 
2 + 3/n + 1/n
+(b − a)2
6
  
2 1 + limn→∞ 1/n
= (b − a) a + 2a(b − a)
2
!!
2 2 + 3 limn→∞ (1/n) + (limn→∞ 1/n)2
+(b − a)
6

() Calculus 1 October 28, 2020 29 / 38


Exercise 5.3.65

Exercise 5.3.65 (continued 4)

Proof (continued).
Z b   
1 + (0)
x 2 dx = (b − a) a2 + 2a(b − a)
a 2
2
 
2 + 3(0) + (0)
+(b − a)2
6
= (b − a) a2 + a(b − a) + (b − a)2 (1/3)


= (b − a)(a2 + ab − a2 + b 2 /3 − 2ab/3 + a2 /3)


= (b − a)(ab/3 + b 2 /3 + a2 /3)
= (ab2 + b 3 + a2 b − a2 b − ab2 − a3 )/3
b 3 a3
= − . 
3 3

() Calculus 1 October 28, 2020 30 / 38


Exercise 5.3.36

Exercise 5.3.36

Exercise 5.3.36. Use Equation (4) (see Exercise 5.3.65) to evaluate the
Z π/2
integral θ2 dθ.
0

Solution. The integrand is f (θ) = θ2 , the lower bound of the integral is


a = 0, and the upper bound of the integral is b = π/2. So by Equation (4)
(Exercise 5.3.65),

π/2
b 3 a3 (π/2)3 03 π3
Z
θ2 dθ = − = − = .
0 3 3 3 3 24

() Calculus 1 October 28, 2020 31 / 38


Exercise 5.3.36

Exercise 5.3.36

Exercise 5.3.36. Use Equation (4) (see Exercise 5.3.65) to evaluate the
Z π/2
integral θ2 dθ.
0

Solution. The integrand is f (θ) = θ2 , the lower bound of the integral is


a = 0, and the upper bound of the integral is b = π/2. So by Equation (4)
(Exercise 5.3.65),

π/2
b 3 a3 (π/2)3 03 π3
Z
θ2 dθ = − = − = .
0 3 3 3 3 24

() Calculus 1 October 28, 2020 31 / 38


Exercise 5.3.18

Exercise 5.3.18
Exercise 5.3.18. Graph Z 0the integrand and use known area formulas to
p
evaluate the integral: 16 − x 2 dx.
−4

Solution.
√ Notice that with y = 16 − x 2 , we have
y 2 = ( 16 − x 2 )2√= 16 − x 2 and y ≥ 0. So x 2 + y 2 = 16 and y ≥ 0. So
the graph of y = 16 − x 2 is the top half (since y ≥ 0) of a circle of
radius r = 4 and center (0, 0):

() Calculus 1 October 28, 2020 32 / 38


Exercise 5.3.18

Exercise 5.3.18
Exercise 5.3.18. Graph Z 0the integrand and use known area formulas to
p
evaluate the integral: 16 − x 2 dx.
−4

Solution.
√ Notice that with y = 16 − x 2 , we have
y 2 = ( 16 − x 2 )2√= 16 − x 2 and y ≥ 0. So x 2 + y 2 = 16 and y ≥ 0. So
the graph of y = 16 − x 2 is the top half (since y ≥ 0) of a circle of
radius r = 4 and center (0, 0):

() Calculus 1 October 28, 2020 32 / 38


Exercise 5.3.18

Exercise 5.3.18
Exercise 5.3.18. Graph Z 0the integrand and use known area formulas to
p
evaluate the integral: 16 − x 2 dx.
−4

Solution.
√ Notice that with y = 16 − x 2 , we have
y 2 = ( 16 − x 2 )2√= 16 − x 2 and y ≥ 0. So x 2 + y 2 = 16 and y ≥ 0. So
the graph of y = 16 − x 2 is the top half (since y ≥ 0) of a circle of
radius r = 4 and center (0, 0):

() Calculus 1 October 28, 2020 32 / 38


Exercise 5.3.18

Exercise 5.3.18 (continued)


Solution.


Since y = f (x) = 16 − x 2 is non-negative, then (by definition) the
Z 0p
definite integral 16 − x 2 dx is the area under the curve
√ −4
y = 16 − x 2 (and above the x-axis) from a = −4 to b = 0. That is, the
integral is 1/4 of the area of
a circle of2 radius r = 4. Therefore,
Z 0p 2
π(r ) π(4)
16 − x 2 dx = = = 4π . 
−4 4 r =4
4
() Calculus 1 October 28, 2020 33 / 38
Exercise 5.3.62

Exercise 5.3.62

Exercise 5.3.62. Graph the function h(x) = |x| and find the average
value over the intervals (a) [−1, 0], (b) [0, 1], and (c) [−1, 1].
Solution. We consider the graph and relevant areas:

() Calculus 1 October 28, 2020 34 / 38


Exercise 5.3.62

Exercise 5.3.62

Exercise 5.3.62. Graph the function h(x) = |x| and find the average
value over the intervals (a) [−1, 0], (b) [0, 1], and (c) [−1, 1].
Solution. We consider the graph and relevant areas:

1
Rb
By definition av(h) = b−a a h(x) dx, so calculating the integrals using
areas we have

() Calculus 1 October 28, 2020 34 / 38


Exercise 5.3.62

Exercise 5.3.62

Exercise 5.3.62. Graph the function h(x) = |x| and find the average
value over the intervals (a) [−1, 0], (b) [0, 1], and (c) [−1, 1].
Solution. We consider the graph and relevant areas:

1
Rb
By definition av(h) = b−a a h(x) dx, so calculating the integrals using
areas we have R0
1
(a) The average of h over [−1, 0] is (0)−(−1) −1 h(x) dx = 1/2 .

() Calculus 1 October 28, 2020 34 / 38


Exercise 5.3.62

Exercise 5.3.62

Exercise 5.3.62. Graph the function h(x) = |x| and find the average
value over the intervals (a) [−1, 0], (b) [0, 1], and (c) [−1, 1].
Solution. We consider the graph and relevant areas:

1
Rb
By definition av(h) = b−a a h(x) dx, so calculating the integrals using
areas we have R0
1
(a) The average of h over [−1, 0] is (0)−(−1) −1 h(x) dx = 1/2 .
1
R 1
(b) The average of h over [0, 1] is (1)−(0) 0 h(x) dx = 1/2 .

() Calculus 1 October 28, 2020 34 / 38


Exercise 5.3.62

Exercise 5.3.62

Exercise 5.3.62. Graph the function h(x) = |x| and find the average
value over the intervals (a) [−1, 0], (b) [0, 1], and (c) [−1, 1].
Solution. We consider the graph and relevant areas:

1
Rb
By definition av(h) = b−a a h(x) dx, so calculating the integrals using
areas we have R0
1
(a) The average of h over [−1, 0] is (0)−(−1) −1 h(x) dx = 1/2 .
1
R 1
(b) The average of h over [0, 1] is (1)−(0) 0 h(x) dx = 1/2 .
(c) The average of h over [−1, 1] is
1
R1
(1)−(−1) −1 h(x) dx = 1/2(1) = 1/2 . 

() Calculus 1 October 28, 2020 34 / 38


Exercise 5.3.62

Exercise 5.3.62

Exercise 5.3.62. Graph the function h(x) = |x| and find the average
value over the intervals (a) [−1, 0], (b) [0, 1], and (c) [−1, 1].
Solution. We consider the graph and relevant areas:

1
Rb
By definition av(h) = b−a a h(x) dx, so calculating the integrals using
areas we have R0
1
(a) The average of h over [−1, 0] is (0)−(−1) −1 h(x) dx = 1/2 .
1
R 1
(b) The average of h over [0, 1] is (1)−(0) 0 h(x) dx = 1/2 .
(c) The average of h over [−1, 1] is
1
R1
(1)−(−1) −1 h(x) dx = 1/2(1) = 1/2 . 

() Calculus 1 October 28, 2020 34 / 38


Exercise 5.3.76

Exercise 5.3.76

Z 1√
Exercise 5.3.76. Show that the value of x + 8 dx lies between
√ 0
2 2 ≈ 2.8 and 3.

Solution. Let f (x) = x + 8 = (x + 8)1/2 . Then
1 1
f 0 (x) = (x + 8)−1/2 = √ and so the only critical point of f is
2 2 x +8
x = −8. So continuous function f has no critical points in [0, 1] and hence
by the technique of Section 4.1, “Extreme Values of Functions on Closed
Intervals,” the extremes of f on [0, 1] occur at the endpoints.

() Calculus 1 October 28, 2020 35 / 38


Exercise 5.3.76

Exercise 5.3.76

Z 1√
Exercise 5.3.76. Show that the value of x + 8 dx lies between
√ 0
2 2 ≈ 2.8 and 3.

Solution. Let f (x) = x + 8 = (x + 8)1/2 . Then
1 1
f 0 (x) = (x + 8)−1/2 = √ and so the only critical point of f is
2 2 x +8
x = −8. So continuous function f has no critical points in [0, 1] and hence
by the technique of Section 4.1, “Extreme Values of Functions on Closed
Intervals,”
p the extremes
√ of f√on [0, 1] occurp at the endpoints.
√ Since
f (0) = (0) + 8 = 8 = 2 2 and f (1) = √ (1) + 8 = 9 = 3, then the
minimum of f on [a, b] = [0, 1] is min f = 2 2 and the maximum is
min f = 3.

() Calculus 1 October 28, 2020 35 / 38


Exercise 5.3.76

Exercise 5.3.76

Z 1√
Exercise 5.3.76. Show that the value of x + 8 dx lies between
√ 0
2 2 ≈ 2.8 and 3.

Solution. Let f (x) = x + 8 = (x + 8)1/2 . Then
1 1
f 0 (x) = (x + 8)−1/2 = √ and so the only critical point of f is
2 2 x +8
x = −8. So continuous function f has no critical points in [0, 1] and hence
by the technique of Section 4.1, “Extreme Values of Functions on Closed
Intervals,”
p the extremes
√ of f√on [0, 1] occurp at the endpoints.
√ Since
f (0) = (0) + 8 = 8 = 2 2 and f (1) = √ (1) + 8 = 9 = 3, then the
minimum of f on [a, b] = [0, 1] is min f = 2 2 and the maximum is
min f = 3.

() Calculus 1 October 28, 2020 35 / 38


Exercise 5.3.76

Exercise 5.3.76 (continued)

Z 1√
Exercise 5.3.76. Show that the value of x + 8 dx lies between
√ 0
2 2 ≈ 2.8 and 3.

Solution (continued). By Theorem 5.2(6), the Max-Min Inequality, we


have
√ Z b
min f · (b − a) = (2 2)((1) − (0)) ≤ f (x) dx
a
Z 1√
= x + 8 dx ≤ max f · (b − a) = (3)((1) − (0),
0
√ Z 1√
of 2 2 ≤ x + 8 dx ≤ 3, as claimed. 
0

() Calculus 1 October 28, 2020 36 / 38


Exercise 5.3.88

Exercise 5.3.88

Exercise 5.3.88. If you average 30 miles/hour on a 150 mile trip and then
return over the same 150 miles at the rate of 50 miles/hour, what is your
average speed for the trip? Give reasons for your answer.

Solution. We define function f (t) as your speed as a function of time t,


where t is measured in hours and f is measured in miles/hour. So we have
f defined piecewise as f (t) = 30 miles/hour for t between 0 hours and 5
hours (since it takes 5 hours to travel 150 miles at 30 miles/hour) and
f (t) = 50 miles/hour for t between 5 hours and 8 hours
 (since it takes 3
30, 0 ≤ t < 5
hours to travel 150 miles at 50 miles/hour): f (t) =
50, 5 ≤ t ≤ 8

() Calculus 1 October 28, 2020 37 / 38


Exercise 5.3.88

Exercise 5.3.88

Exercise 5.3.88. If you average 30 miles/hour on a 150 mile trip and then
return over the same 150 miles at the rate of 50 miles/hour, what is your
average speed for the trip? Give reasons for your answer.

Solution. We define function f (t) as your speed as a function of time t,


where t is measured in hours and f is measured in miles/hour. So we have
f defined piecewise as f (t) = 30 miles/hour for t between 0 hours and 5
hours (since it takes 5 hours to travel 150 miles at 30 miles/hour) and
f (t) = 50 miles/hour for t between 5 hours and 8 hours
 (since it takes 3
30, 0 ≤ t < 5
hours to travel 150 miles at 50 miles/hour): f (t) =
50, 5 ≤ t ≤ 8

() Calculus 1 October 28, 2020 37 / 38


Exercise 5.3.88

Exercise 5.3.88 (continued)



30, 0 ≤ t < 5
Solution (continued). . . . f (t) = So, by definition,
50, 5 ≤ t ≤ 8
the average speed (i.e., the average of f over [0, 8]) is
Z b Z 8
1 1
av(f ) = f (t) dt = f (t) dt
b−a a (8) − (0) 0
Z 5 Z 8 
1
= f (t) dt + f (t) dt by Theorem 5.2(5), Additivity
8 0 5
Z 5 Z 8 
1
= 30 dt + 50 dt
8 0 5
1
= ((30)((5) − (0)) + (50)((8) − (5))) by Exercise 5.3.63
8
1 300 75
= (150 + 150) = = miles/hour.
8 8 2


() Calculus 1 October 28, 2020 38 / 38
Exercise 5.3.88

Exercise 5.3.88 (continued)



30, 0 ≤ t < 5
Solution (continued). . . . f (t) = So, by definition,
50, 5 ≤ t ≤ 8
the average speed (i.e., the average of f over [0, 8]) is
Z b Z 8
1 1
av(f ) = f (t) dt = f (t) dt
b−a a (8) − (0) 0
Z 5 Z 8 
1
= f (t) dt + f (t) dt by Theorem 5.2(5), Additivity
8 0 5
Z 5 Z 8 
1
= 30 dt + 50 dt
8 0 5
1
= ((30)((5) − (0)) + (50)((8) − (5))) by Exercise 5.3.63
8
1 300 75
= (150 + 150) = = miles/hour.
8 8 2


() Calculus 1 October 28, 2020 38 / 38

You might also like